Đến nội dung

Khongten012

Khongten012

Đăng ký: 01-09-2016
Offline Đăng nhập: 09-01-2017 - 17:46
-----

Trong chủ đề: TÌm min P $\frac{x^3}{(y+z)^2}+\frac...

02-01-2017 - 08:20

attachicon.gifCodeCogsEqn (2).gif

Lại có $\frac{1}{x}+\frac{1}{y}+\frac{1}{z}\geq \frac{9}{x+y+z}$

Mà $\frac{1}{x}+\frac{1}{y}+\frac{1}{z}\leq 3$

suy ra $x+y+z\geq 3$

Vậy min P =$\frac{3}{4}$ khi x=y=z=1

bước 4 làm sao để chuyển sang bươc 5 được như thế????


Trong chủ đề: TÌm min P $\frac{x^3}{(y+z)^2}+\frac...

02-01-2017 - 08:19

Từ giả thiết bạn chứng minh $x+y+z\geq 3$

$\sum \frac{x^{3}}{(x+y)^{2}}\geq \frac{1}{2}\sum \frac{x^3}{x^2+y^2}=\frac{1}{2}\sum x-\frac{1}{2}\sum \frac{xy^{2}}{x^2+y^2}\geq\frac{1}{2} \sum x-\frac{1}{4}\sum \frac{xy^{2}}{xy}=\frac{1}{4}\sum x\geq \frac{3}{4}$

Dấu bằng xảy ra khi x=y=z=1

bạn ơi bạn làm sai đề rồi


Trong chủ đề: giải hệ phương trình : x^{2}+y^{2}+\frac{8x...

19-10-2016 - 20:35

http://diendantoanho...-học-2016-2017/

Bạn vào link này có nha


Trong chủ đề: Hỏi có thể khẳng định mỗi số trong 2013 số đã cho lớn hơn 3000 hay không?

16-10-2016 - 10:57

Có 
sắp xếp 2013 số đó đi cho tổng của 1007 số bé nhất > tổng của 1006 số còn lại + 2012
đáp số phải là mỗi số > 2012 + 10062

bạn có thể giải thích rõ hơn không?